44
$\begingroup$

It follows from a general theorem of Honda that the formal group with the logarithm $$ x+x^{2^s}/2+x^{3^s}/3+x^{4^s}/4+\cdots $$ has integer coefficients. I became interested in it because its $p$-typizations give the formal groups of the $s$th Morava K-theories (after reducing modulo $p$).

In particular I wonder whether the series $$ \sum_{n\geqslant1}\frac{x^{n^2}}n $$ which one obtains for $s=2$ is related in any way to modular forms and elliptic curves.

Does anybody know where to find information about this?

P.S. - Decided to add a picture: here is the color-coded modulus of the derivative of the above series as a function of a complex variable $x$ in the unit disk, where its ``modular-like'' behavior is especially apparent.

enter image description here

P.P.S. - ...and for some further suspense, here are the first few terms of the formal group itself. Notation: $s$ is the sum of the two variables and $p$ is their product. Note the reappearing factors. \begin{align*} s\\ -p&(2s^2-p)\\ +2s^3p&(2s^2-p)\\ -sp&(3s^6-9s^4p+10s^2p^2-3p^3)\\ -s^2p&(2s^2-p)(4s^4+6s^2p-3p^2)\\ +s^4p&(12s^6-21s^4p+20s^2p^2-6p^3)\\ +2sp&(2s^2-p)(4s^8+18s^6p-5s^4p^2-4s^2p^3+p^4)\\ -2s^3p&(18s^{10}+18s^8p-67s^6p^2+87s^4p^3-48s^2p^4+9p^5)\\ -s^2p&(36s^{12}+246s^{10}p+72s^8p^2-493s^6p^3+356s^4p^4-106s^2p^5+12p^6)\\ +3s^9p&(3s^6-9s^4p+10s^2p^2-3p^3)\\ +... \end{align*}

$\endgroup$
8
  • 1
    $\begingroup$ I don't know. $\sum x^{n^2}$ is a theta-function, and I know that one is very closely related to modular forms. $\endgroup$ Nov 3, 2013 at 5:12
  • 6
    $\begingroup$ I'm not an expert, but the Eichler integral can be defined, purely formally, as the operator that sends, with your notation, $x^n \mapsto x^n/n$ (recall that for modular forms $x=exp(2\pi i z)$, so this is integration $dz$). Your series is then a formal ``half-integral'' of the standard theta function. I've seen such things arise in talks, though I don't know anything about them myself (and I trust your ability to google as much as my own). I'd recommend searching for "half-derivative" rather than "half-integral", since the latter appears quite frequently with another meaning. $\endgroup$
    – rlo
    Nov 3, 2013 at 18:34
  • 5
    $\begingroup$ I still do not have anything definite to say; however, thanks to @rlo I found a really amazing identity: in link Andrews, Jiménez Urroz and Ono have found, generalizing previous amazing discoveries of Andrews and Zagier, among other things, an identity equivalent to the following: $$ 4\sum_{n\geqslant1}nq^{n^2}=\left(\sum_{n\in\mathbb Z}q^{n^2}\right)\left(2\sum_{n\geqslant1}\frac{q^{2n-1}}{1+q^{2n-1}}+ \sum_{n\geqslant1}\left(1-\prod_{k\geqslant n}\left(\frac{1+q^k}{1-q^k}\right)^{(-1)^k}\right)\right). $$ Isn't this crazy!? $\endgroup$ Nov 9, 2013 at 17:26
  • 4
    $\begingroup$ Here is another very interesting version. It does not come any closer to the goal for me, but still... Let $$ \theta_{\geqslant n}(q):=\prod_{k\geqslant n}\left(\frac{1-q^k}{1+q^k}\right)^{(-1)^k}; $$ in particular $\theta_{\geqslant1}=\theta$ is the "usual" theta function. Then, $$ 2\sum_{n\geqslant1}nq^{n^2}=\theta(q)\sum_{n\geqslant1}\frac{\theta_{\geqslant n}(q)-\theta_{\geqslant n}(q)^{-1}}2. $$ Cool, isn't it? :) $\endgroup$ Nov 10, 2013 at 10:07
  • 2
    $\begingroup$ This looks a little bit similar to my own question: mathoverflow.net/questions/320070 $\endgroup$
    – F. C.
    Apr 2, 2021 at 8:40

2 Answers 2

5
$\begingroup$

The following answer is not really satisfactory for me; however it seems to be the analog of current results on similar phenomena like partial, mock and quantum modular forms, so I decided to post it here in hope that somebody will contribute further improvements.

Using help from another question I posted later on, I can now claim this:

let $\tilde\theta(\tau):=\sum_{n\geqslant1}ne^{n^2\pi i\tau}$ be (up to a constant) the derivative wrt $\tau$ of the series in question with $x=e^{\pi i\tau}$; then in the upper half-plane, $$ \tilde\theta(-1/\tau)=(i\tau)^\frac32\tilde\theta(\tau)-\frac{i\tau}\pi\int\limits_0^\infty t\coth(\sqrt{\pi i\tau}t)e^{-t^2}dt. $$ The last term must be closely related to the Mordell integral; for large $z=i\tau/\pi$ its asymptotic behavior is given by the (divergent) series $$ \frac12\sum_{n\ge0}\frac{B_{2n}}{n!}z^{1-n}=\frac z2+\frac1{12}-\frac1{120z}+\frac1{504z^2}-\frac1{1440z^3}+\frac1{3168z^4}-\frac{691}{3931200z^5}+... $$ which somehow explains the near-modular features of $\tilde\theta$. I think I will post a followup question to clarify relationship with some recent work mentioned by @rlo in a comment above.

Another thing I do not understand well: it seems that I cannot extend the first equality analytically simultaneously to both branches of the square root.

$\endgroup$
0
$\begingroup$

Again in no sense a complete answer - I just found a partial explanation for my very last remark about reappearing factors in the homogeneous summands of the formal group.

Let $L_s(x)$ be the above logarithm, i. e. $\sum_{n\geqslant1}\frac{x^{n^s}}n$, and let $F_s(x_1,x_2,...)$ be the (many variable variant) of the corresponding formal group, i. e. $F_s=E_s(\sum_iL_s(x_i))$ where $E_s$ is the compositional inverse of $L_s$).

Adapting to our situation the argument for Theorem 4.3.9 (page 141, or page 121 of the online version) of Ravanel's Green Book one finds$$F_s(x_1,x_2,...)=F_s(w_1,w_2^{2^{s-1}},...,w_n^{n^{s-1}},...)$$where $w_n$ are the Witt symmetric functions (determined by $\sum_ix_i^n=\sum_{d|n}dw_d^{\frac nd}$).

Thus in particular for $s=2$ we get\begin{multline*}F_2(x,y)=F_2(w_1(x,y),w_2(x,y)^2,w_3(x,y)^3,...)\\=F_2(x+y,(-xy)^2,(-xy(x+y))^3,(-xy(x+y)^2)^4,(-xy(x^3+x^2y+xy^2+y^3))^5,...)\end{multline*}which tells us that there will be lots of reappearing factors in the expansion...

I've asked a new question Efficiently computing (plethysm-like?)substitutions of symmetric functions about handling tricky expressions like that for $F_s$ above.

$\endgroup$

Your Answer

By clicking “Post Your Answer”, you agree to our terms of service and acknowledge you have read our privacy policy.

Not the answer you're looking for? Browse other questions tagged or ask your own question.